Advanced Pharm Final Questions, Katzung pharmacology chap1 questions, Pharm II - MCQs, Pharm ABX : Overview + Penicillin
5. Ampicillin is eliminated by first-order kinetics. Which of the following statements best describes the process by which the plasma concentration of this drug declines? (A)There is only 1 metabolic path for drug elimination (B)The half-life is the same regardless of the plasma concentration (C)The drug is largely metabolized in the liver after oral administration and has low bioavailability (D)The rate of elimination is proportional to the rate of administration at all times (E)The drug is distributed to only 1 compartment outside the vascular system
"First-order" means that the elimination rate is proportional to the concentration perfusing the organ of elimination. The half-life is a constant. The rate of elimination is proportional to the rate of administration only at steady state. The order of elimination is independent of the number of compartments into which a drug distributes. The answer is B.
Dosing of natural penicillins? What is unique about the dosing scheme?
.6micrograms of crystalline sodium salt of PCN G It is measured as international units and must be convereted
Narrow spectrum penicillins, both penicillinase sensitive and penicillinase resistant, have relatively poor activity against gram negative bacteria. What is the main property or characteristic that explains why these microorganisms to not respond well to the penicillins? A. Actively transport any absorbed PCN back to the extracellular space B. Have an outer membrane that serves as a physical barrier to the PCN C. Lack of a surface enzyme necessary to metabolically citrate the PCN D. Lack PBPs E. Metabolically inactivate these PCN by mechanisms not involving beta lactamases
*B. Have an outer membrane that serves as a physical barrier to the PCN* Both susceptible and resistant Gram Negative and Gram Positive organisms have PBP resistance to narrow spectrum PCNs due to the presence of outer membrane with pores too small to allow adequate penetration of the drug and access to the PBPs. C. Lack of surface enzyme necessary to metabolically activate the PCN :: IS WRONG b/c PCN is already active.
Agents that target the cell wall (3 major groups and several in one of the groups)
*Beta Lactams* Cephalosporins Penacillins Carbopenems Monobactams *Glycopeptides* Vancomycin *Cyclic Glycopeptide* Daptomycin
*Beta lactamase inhibitors* Use? Irreversibly binds____ Most active against______ such as___, ___. __, __ Not good against ______ such as __, __, __
*Beta lactamase inhibitors* Weak intrinsic antimicrobial activity Irreversibly binds beta lactamases Most active against plasmid encoded : -Gonococci, Streptococci, E. Coli, H. Influenza Not good against inducible chromosomal: -Enterobacter, Pseudomonas, Serratia
*Beta lactamase inhibitors* When added to penicillins.... You get coverage for ___ and _____ Clinical Role : __,_____, ___________, ____ ________has good activity against some Extended spectrum beta lactamases
*Beta lactamase inhibitors* When added to penicillins---- You get coverage for gram negatives and anaerobes Clinical role in OM, Cellulitis, Low-risk febrile neutropenia, and sinusitis *Tazobactam* has good activity against some extended spectrum beta lactamases
A sputum culture of a 65yo man with pneumonia is positive for beta lactamase positive staphylococci. Which of the following is the best choice for penicillin therapy in this patient? A. Ampicillin B. Carbenicillin C. Oxacillin D. Penicillin G E. Ticarcillin
*C. Oxacillin* The rest are wrong due to the fact that is a beta lactamase positive staph, Ampicillin is not resistant to this, carbenicillin does not have any notable resistance to this, Pen G has problems with beta lactamase,
Agents such as clavulonic acid, sulbactam, or tazobactam are often added to some proprietary penicillin combination products. What is the main reason for including them, or describes their action best? A. Add antibiotic activity against Pseudomonas and many Enterobacter species. B. Facilitate antibiotic penetration into central nervous system and cerebrospinal fluid. C. Inhibit cell wall transpeptidases. D. Inhibit inactivation of penicillin by beta lactamase producing bacteria. E. Inhibit the normally significant hepatic metabolism of the penicillin F. Reduce the risk and/or severity of allergic reactions in susceptible patients.
*D. Inhibit inactivation of penicillin y beta lactamase producing bacteria*
A patient with P. Aeruginosa infection is receiving intravenous gentamicin. The amino glycoside blood concentrations are well above the MIC but the clinical response is not satisfactory, a new medication order calls for adding a penicillin, administered in separate IV lines to avoid a physical incompatibility. *If this order is carried out, what is most likely to occur?* A. The AMG will inactivate the PCN B. The AMG will chemically neutralize and abolish the effects of the PCN C. The patient is likely to develop C. Difficule colitis (superinfection) D. The PCN will act synergistically with the AMG E. The PCN will increase risk of AMG nephrotoxicity F. The risk of inducing resistance to both drugs increases dramatically
*D. The PCN will act synergistically with the AMG* Penicillin weaken cell walls, facilitating access of AMG to site of action (Protein synthesis inhibited) better antibiotic response than either alone.
MOA of penicillin?
*Inhibit cell wall* -Bind to specific enzymes (PBP) within cytoplasmic membrane (covalently) -Inhibit the transpeptidation reaction that x-links the linear peptidoglycan chains -Activate autolytic enzymes that cause lesion in bacterial cell wall.
What are the 3 or 4 toxicities we need to know about with Beta lactam ABX?
1. Allergic reactions (Hives, Rash) not familial! 2. Nephrotoxicity 3. BM suppression 4. Diarrhea (Probably number one cause of pseudomemranous colitis)
*antistaph pens* 1. this drug is better for renal dysfunction patients 2. This drug can cause hepatitis
1. Nafcillin (primarily liver metabolized) 2. Oxacillin
Special features about Pen G ____ absorbed from intestinal tract ______ rapidly destroys it IV : Peaks in ______minutes, half life of ______ PO : Max [] is ______minutes, ______ interferes *special features about Pen G Benzathine* Released_____ from injection site Duration of _______days
1/3 absorbed from intestinal tract Gastric acids rapidly destroy it IV: Peaks in 15-30 minutes, with a half life of 30 min. PO : max [] is 30-60 minutes, food interferes Pen G Benzathine: Released slowly from injection site Duration of 26 days
How many types of PBPs? Why would you not want to use multiple beta lactams together?
3 types of PBPs - different affinity based on drug, but the receptors are saturable, so using multiple beta lactams together is not recommended - too similar in MOA
65 year old woman with PNA was given tobramycin, 150mg IV. After 20min, plasma concentration measured and found to be 3mg/L. Assuming no elimination of drug in 20 minutes, what is apparent volume of distribution of tobramycin?
50L
Which one of the following drugs or drug classes, whether available OTC or rx, is most likely to relieve acid-related symptoms the fastest when taken at usually effective/recommended doses? A. Antacids B. Cimetidine C. H2 blockers D. Misoprostol E. PPIs
A. Antacids
A very worried patient who had self-medicated for diarrhea, calls you and tells you his tongue is black and his stools have darkened, too. Which drug most likely caused these signs? A. Bismuth salts B. Calcium carbonate C. Cimetidine D. Omeprazole E. Magnesium-aluminium antacid combination
A. Bismuth salts
Which of the following statements about fentanyl is correct? A. Fentanyl is 100 times more potent than morphine. B. Its withdrawal symptoms can be relieved by naloxone. C. The active metabolites of fentanyl can cause seizures. D. It is most effective by oral administration.
A. Fentanyl is 100x more potent than morphine.
30-A 54 year old contractor complains of anginal pain that occurs at rest. On exam, his BP is 145/90 and HR 90. A treatment of angina that often decreases the heart rate and can prevent vasospastic angina attacks is
A Diltiazem
29-A 70 year old man has severe urinary hesitancy associated with benign prostatic hyperplasia. He has tried alpha blockers with little relief. His physician recommends a drug that blocks 5alpha reductase in the prostate. Which of the following drugs would the physician most likely prescribe?
A Finasteride
50-Although fentanyl or one of its cogeners is usually administered in the early stages of a general anesthesia procedure, it is likely that the patient will receive an injection of morphine during the last phase. The rationale for switching to morphine is that the drug has
A a longer duration of action
76-Chronic heart failure is commonly treated with a combination of drugs that improve symptoms and may provide long term survival benefits. Three drugs or drug groups that have been shown in clinical trials to provide survival benefits in patients with chronic heart failure are
A ace inhibitors, carvedilol, and spironolactone
27-Which of the following agents is most likely to be of immediate therapeutic value in relieving an acute bronchospastic attack?
A albuterol
36-Which of the following drugs often causes tachycardia and tremor when used in asthma?
A albuterol
78-Which vasodilator acts on smooth muscle to block calcium influx via L type channels?
A amlodipine
98-JW is 78 year old man diagnosed with deep vein thrombosis. The patient has a history of heparin induced thrombocytopenia. Which of the following is the most appropriate drug for parenteral administration in this patient?
A argatroban
38-A 30 year old patient in the cardiac intensive care unit is to receive a beta antagonist drug. Typical responses to beta receptor blockade include
A bradycardia
35-A 15 year old presents in the asthma clinic for evaluation of her medications. She has had no hospitalizations for severe, refractory asthma but episodes have occurred at night and have caused absences from school. Which of the following combinations is most appropriate for the prophylactic management of her asthma?
A budenoside plus formoterol by inhalation once or twice daily
81-In a patient with familial combined hyperlipidemia that is associated with increased VLDL and LDL, which of the following drugs is most likely to increase plasma TG while also decreasing plasma LDL?
A cholestyramine
34-the cellular cause of digitalis toxicity is
A intracellular calcium overload
Diaphragm is breakpoint for what bug classes
Above diaphragm is gram + anaerobes, below diaphragm is gram - anaerobes
45-A young patient has a seizure disorder with recurrent contractions starting in muscles of the right hand that spread through the arm and the right side of the face. The attacks last for only a minute or two with no loss of consciousness. Which of the following drugs is least likely to be useful in the treatment of this patient?
B ethosuximide
Which of the following drugs slows conduction thru AV node and has duration of action 10-20 seconds?
Adenosine
· Though rare, a serious toxicity associated with the thioamides is which of the following?
Agranulocytosis rarely, the thioamides cause severe adverse reactions including agranulocytosis, vasculituis, and hepatic damage.
When does the nephrotoxicity associated with penicillins really become a big deal?
Alone they are sort of meh, but they have a hidden nephrotox that comes out when administered with other nephrotox drugs like Vanco, NSAIDS, Radionucleotide, Dye,
• You are leading Grand Rounds with a cohort of NP students, and presenting a patient who is admitted for recurrent near syncopal episodes. You suspect the patient is having challenges with postural hypotension. You teach your colleagues that postural hypotension is a common adverse effect of which one of the following types of drugs.
Alpha receptor blockers
97-A 35 year old man presented with lethargy, weight gain, and muscle weakness. Lab findings confirm the diagnosis of hypothyroidism. In the treatment of hypothyroidism, thyroxine is preferred over liothyronine because thyroxine
B has longer half life
A patient new to your practice has recurrent bacterial sinusitis. She reports antibiotics at least three times a year none of which ever seem to work the first time. She is a poor historian, and cannot remember the names of what she used in the past. She has recently failed treatment with another antibiotic. Because of her past use of antibiotics, select a broad coverage antibiotic to treat her chronic sinus infection.
Amoxicillin-clavunate (Augmentin)
Name the aminopenicillins
Amoxillin Ampicillin
A 19 year old being treated for leukemia develops a fever. You give several agents that will cover bacterial, viral, and fungal infections. Two days later, he develops acute renal failure. Which of the following drugs is most likely responsible?
Amphoterericin B
Every baby in the hospital gets what two antibiotics just in case they have sepsis?
Ampicillin and Gentomycin
4. A 12-year-old child has bacterial pharyngitis and is to receive an oral antibiotic. Ampicillin is a weak organic acid with a pKa of 2.5. What percentage of a given dose will be in the lipid-soluble form in the duodenum at a pH of 4.5? (A)About 1% (B)About 10% (C)About 50% (D)About 90% (E)About 99%
Ampicillin is an acid, so it is more ionized at alkaline pH and less ionized at acidic pH. The Henderson-Hasselbalch equation predicts that the ratio changes from 50/50 at the pH equal to the pKa to 1/10 (protonated/unprotonated) at 1 pH unit more alkaline than the pKa and 1/100 at 2 pH units more alkaline. For acids, the protonated form is the nonionized, more lipid-soluble form. The answer is A.
In phase 2 clinical trial in hypertensive patients, an endogenous octapeptide vasoconstrictor was found to increase in the blood of patients treated with large doses of diuretics. Which of the following is the most likely endogenous peptide?
Angiotensin II
· It is contraindicated to use which respiratory agent with patients that have a soy and peanut allergy?
Anticholinergics
Concerned with antibiotics What does concentration dependent imply?
Antimicrobial activity is directly impacted by : *how high Cmax is above the MIC* _need 10x_ (Vanc)
63-AT, a 23 year old woman with severe asthma, has been taking oral prednisone to reduce her asthma attacks. She is encouraged when you tell her about budesonide, an inhaled steroid. The topical administration and short half life of budesonide greatly reduce the risk of systemic side effects compared with oral prednisone. The long term daily oral administration of therapeutic doses of prednisone results in which of the following adverse effects?
B decreased bone density
86-Drugs classified as selective serotonin reuptake inhibitors have minimal clinical efficacy in the treatment of patient who suffer from
B diminished sexual function and interest
15-A 15 year old girl is admitted complaining of palpitations and shortness of breath. An ECG reveals sinus tachycardia with a heart rate 160. A drug suitable for producing a brief (5-15min) increases in vagal effects on the heart is
B edrophonium
In considering your selection of an antibiotic, you must first consider a full understanding of the potential bacteria. You know that bacteria are savvy, and remarkably adaptable, and have the ability to continually alter their genetic makeup. How do bacteria alter their genetic makeup?
Any of these methods of transfer may happen.
Recurrent otitis media what is the DoC?
Augmentin (Ampicillin + Clavulonic acid?)
What is an example of a drug that has a long half life and has an extended PAE?
Azithromycin
27-An accepted clinical use of antimuscarinic drugs is for treatment of
B COPD
28-DF is a 58 year old man who just underwent a stent placement procedure. He is discharged on dual antiplatelet therapy. One of the drugs is aspirin, while the second drug targets the ADP receptor. Which of the following is a prodrug, the metabolite of which binds to and inhibits the platelet ADP receptor?
B Clopidigrel
31-Which of the following drugs is partial agonist that can cause vasoconstriction in the absence of other drugs?
B Ergotamine
15- A 72 year old man with atrial fibrillation required oral anticoagulation to reduce his risk of ischemic stroke. He was started on rivaroxaban. What is the molecular target of rivaroxaban?
B Factor Xa
8-While on vacation, a 35 year old man with a 10 year history of myasthenia gravis loses his supply of medications. He is now admitted to the emergency department complaining of diplopia, dysarthria, and difficulty swallowing. The most appropriate drug from the following list for reversing myasthenia crisis in this patient is
B Neostigmine
58-Ramelteon, a drug prescribed for insomnia, is thought to act in the CNS via
B activation of melatonin receptors
87-A 29 year old accountant has recurrent episodes of tachycardia with ECG characteristics of torsades de pointes. Which of the following drugs is likely to reduce these episodes?
B amiodarone
61-The antibacterial action of aminoglycosides is due to their ability to
B bind to the 30s ribosomal subunit and block initiation of bacterial protein synthesis
24-A 55 year old executive has hypertension and cardiac hypertrophy. He is being treated with diuretics. The mechanism of action is chlorthalidone is described as
B blockade of NA/CL cotransporter
36-JP is 47 year old woman with type 2 DM. She is taking metformin, glipizide, and pioglitazone. In spite of her medications, the HbA1c is 8% You propose to add a new medication that acts in the kidney to lower her HbA1C to less than 6.5%. Which of the following oral antidiabetic drugs should you add?
B canagliflozin
81-A 66 year old patient is diagnosed with hypertension and angina. A drug with benefits in both conditions is suggested. Which of the following drugs has both nonselective beta blocking and alpha 1 selective blocking action?
B carvedilol
20-A 40 year old woman was being treated for chronic moderate hypertension. When she went on vacation and forgot her pills, her BP rose markedly and she was admitted to the emergency service with blurred vision, severe headache, and retinal hemorrhages. Her BP was found to be 200/120. A drug that is most likely to be followed by severe rebound hypertension if stopped suddenly is
B clonidine
23-A 54 year old farmer is brought to the emergency department in a coma. Blood assay reveals severe hypercalcemia. Which of the following is the most appropriate immediate therapy for this man?
B loop diuretic plus saline infusion
16-A patient with 30 year history of type 1 DM comes to you with a complaint of bloating and sour belching after meals. On several occasions, vomiting has occurred after a meal. Evaluation reveals delayed emptying of the stomach, and you diagnose diabetic gastroparesis. Which drug would be most useful in this patient?
B metoclopramide
An opioid abuser, seeking something to self-administer for subjective responses, gets a large amount of diphenoxylate and consumes it all at once. He is not likely to do this again because he has consumed a combination product that not only contains the diphenoxylate but also another drug that causes a host of unpleasant systemic responses. What is the other drug found in combination with the diphenoxylate? A. Morphine B. Atropine C. Ipecac D. Magnesium sulfate E. Naltrexone
B. Atropine
A pt undergoing cancer chemo gets ondansetron for prophylaxis of drug-induced n/v. Which best describes this drug's main mechanism of action in this setting? A. Activates mu-type opioid receptors in the CTZ B. Blocks central serotonin (5-HT3) receptors C. Blocks dopamine receptors D. Blocks histamine H1 receptors in the brainstem and inner ear E. Suppresses gastric motility and acid secretion via muscarinic blockade
B. Blocks central serotonin (5-HT3) receptors
A patient with multiple medical problems is taking several drugs, including theophylline, warfarin, quinidine, and phenytoin. The pt suffers some GI distress and starts taking a new drug recommended to him by one of his friends. He presents with excessive or toxic effects from all of his other medications and blood tests reveal the plasma conc of all of the rx drugs are high. Which drug did the pt most likely self-rx and take? A. Antacid B. Cimetidine C. Famotidine D. Nizatidine E. Ranitidine
B. Cimetidine Cimetidine is a potent inhibitor of CYP450 (2C9 and 3A4)
In already immunosuppressed patients, what othe rarely reported toxicities do you know for penicillin administration? Which drug are you tinting about right now?
BM depression, granulocytopenia Hepatitis Changes in hemostatis -Impaired platelet aggregation -Interference of binding aggregating agents to platelet receptors OXACILLIN
Your emergency room patient is being admitted to intensive care for multiple system illness. It is suspected he has aggressive infection. You know your treatment plan should include which category of drugs?
Bactericidal antibiotics
52-After ingestion of a meal that included sardines, cheese, and red wine, a patient taking phenelzine experienced an increase in BP 187/133 He was taken to the emergency department. The most likely explanation for this untoward effect is that phenelzine
C inhibits metabolism of catecholamines
Site of action of penicillinase
Beta lactam ring (breaks C-N bond)
7-A 37 year old female presented with epigastric pain for the last 3 days and two episodes of bilious vomiting. Four years earlier, she presented with similar complaints and was diagnosed as having acute pancreatitis with type 2DM, HTN, and combined dyslipidemia (elevated LDL and VLDL). Treatment was started with insulin, telmisartan, atorvastatin, and a low fat diet. Which of the following additional drugs is required to fully treat her dyslipidemia?
C Gemfibrozil
16-A 55 year old man with a strong family history of cardiovascular disease has a history of moderate HTN and angina pectoris. At the clinic today, his BP 160/109 and the ECG shows left ventricular hypertrophy. The rest of his physical examination and labs are normal. His angina is precipitated by exercise. You have been asked to recommend a drug regimen for both conditions. The antihypertensive drug most likely to aggravate angina pectoris is
C Hydralazine
3-A 57 year old man is in the coronary unit after an acute myocardial infarction. He has developed signs of heart failure with reduced ejection fraction of rapidly increasingly severity and several drugs have been suggested. Furosemide, dobutamine, and digoxin can each
C Reduce pulmonary edema
25-A 56 year old woman has arthritis of the knees that limits her activity and post herpetic neuralgia on her torso after an episode of herpes zoster. Which of the following drugs is used topically to control arthritic pain and post herpetic neuralgia?
C capsaicin
22-Metoprolol and minoxidil have which of the following effects in common?
C decreased MAP
31-A 34 year old woman in her second trimester of pregnancy presented with a tender, red, swollen calf that was diagnosed as deep vein thrombosis (DVT) and was treated successfully. However, because of the high risk of recurrence of the DVT, she was treated with an anticoagulant for the remainder of her pregnancy. The drug used most likely was which of the following ?
C enoxaparin
57-Several drugs used in patient with advanced Parkinsons disease allow patients to lower their dose of L dopa/carbidopa, and thus reduce the incidence of L dopa induced dyskinesias. These drugs also decrease the amount of "Off" time for the patient. Which drug is used in this way but does not, if used alone, ameliorate the symptoms of early Parkinsons disease or enhance CNS dobaminergic activity?
C entacapone
79-A 60 year old man with a history of mild myocardial infarction has the following lipid panel. Serum HDL cholesterol 33; serum TG 160; total serum cholesterol 150 Which of the following drugs is likely to result in the correction of this patients dyslipidemia?
C gemfibrozil
97-Which of the following drugs is most likely to cause hypoglycemia when used as monotherapy in the treatment of a patient with type2 DM?
C glipizide
37-Angioneurotic edema precipitated by exposure to an allergen can cause life-threatening laryngeal edema. Two of the probable mediators of such reactions are
C histamine and bradykinin
41-A 57 year old contractor with hypertension has been treated by 2 different physicians. He now comes to the emergency department with a severe reaction. Questioning reveals that he has been taking captopril and spironolactone. This combination is usually ill advised because of the risk of
C hyperkalemia
100-A 55 year old woman came in for routine visit. On exam her height is 1.7M (5'7") , weight is 115kg (253lb) (BMI 40 normal BMI 19-24) BP 135/75 and HbA1C value is 8%. You propose to start her on metformin. Before initiating therapy, it is important to confirm that the patient has normal renal function because patients with unrecognized renal insufficiency who take normal doses of metformin are at increased risk of which of following?
C lactic acidosis
46-A 68 year old man with metastatic prostate cancer is suffering from pain that is gradually increasing in intensity and levorphanol (a strong mu receptor agonist) is prescribed. With chronic use of the drug, tolerance is not likely to develop to constipation or
C pupillary constriction
28-A long acting beta 2 selective agonist that is used as an inhaled prophylactic therapy for moderate or severe asthma is
C salmeterol
A 76-year-old female with renal insufficiency presents to the clinic with severe pain secondary to a compression fracture in the lumbar spine. She reports that the pain has been uncontrolled with tramadol, and it is decided to start treatment with an opioid. Which of the following is the best opioid for this patient? A. Meperidine. B. Fentanyl transdermal patch. C. Hydrocodone. D. Morphine.
C. Hydrocodone would be the best choice of the opioid given in this case. It will be very important to use a low dose and monitor closely for proper pain control and any side effects. Meperidine should not be used for chronic pain, nor should it be used in a patient with renal insufficiency. The transdermal patch is not a good option, since at this time, her pain would be considered acute and she is opioid naïve. Morphine also is not the best choice in this case due to the active metabolites that can accumulate in renal insufficiency.
An elderly woman with a recent history of myocardial infarction is seeking a medication to help treat her occasional heartburn. She is currently taking several medications, including aspirin, clopidogrel, simvastatin, metoprolol, and lisinopril. Which of the following choices should be avoided in this patient? A. Calcium citrate. B. Famotidine. C. Omeprazole. D. Ranitidine. E. Calcium carbonate.
C. Omeprazole may possibly decrease the efficacy of clopidogrel because it inhibits the conversion of clopidogrel to its active form.
You encounter a pt who is taking a PPI, bismuth subsalicylate, metronidazole and tetracycline. What is the most likely condition for which this drug combination is being used? A. Antibiotic-associated pseudomembranous colitis B. IBS C. Refractory or recurrent, and severe, gastric or duodenal ulcers secondary to H. pylori D. "Travelers' diarrhea," severe, E. coli-induced from drinking contaminated water. E. Ulcers that occur in response to long term, high-dose, NSAID therapy for arthritis
C. Refractory or recurrent, and severe, gastric or duodenal ulcers secondary to H. pylori
Which of the following drugs would be the most effective anti-motion sickness drug for a person planning to go on a cruise? A. Atropine. B. Tropicamide. C. Scopolamine. D. Darifenacin. E. Tiotropium.
C. Scopolamine
Which of the following is an accepted therapeutic indication for the use of antimuscarinic drugs?
COPD
Jane is a postal worker recently exposed to a powder of unknown origin. Her post office just underwent a hazardous terrorist scare about anthrax. Jane is mandated to be treated with ciprofloxacin. You know that drug-drug interactions are a big worry. You instruct Jane to stop taking what supplement while she is on ciprofloxacin
Calcium supplement
Why is PAE (post ABX Effect) important?
Can't find a concentration in blood stream, but it is still working, can wait to give another dose of the drug
*Extended Spectrum Pens* Carboxypenicillins (drug we consider here?) Has hadded stability to which organisms? High concentration of which ion? Interferes with what function?
Carboxypens we think of Ticarcillin Had added susceptibility to gram negatives High concentrations of sodium Interferes with the function of platelets
Main antibiotic targets (6)
Cell wall Cell membrane DNA/RNA synthesis Folic acid inhibitors 30s 50s ribosomes
Largest class of antibacterials?
Cell wall inhibitors
A patient's history has a documented severe anaphylactic reaction to penicillin. What other antibiotic or class is likely to cross-react and should be avoided in this patient?
Cephalosporin
PEN nucleus?
Chief structural requirement for biologic activity transformation causes loss of antibacterial activity
· Your patient is struggling with multiple comorbid issues. He is especially angry and embarrassed since he is developing erectile dysfunction. He swears its all your fault since you recenty prescribed several new drugs. Because of a recent diagnosis of GERD, which H2 Antagonist, might you have prescribed that is noted to have the adverse drug reaction of loss of libido, nausea, gynecomastia, impotence?
Cimetidine (Tagamet®)
The three beta lactamase inhibitors?
Clavulanate Tazobactam Sulbactam
Clavulanic acid is combined with what two drugs to make what combinations?
Clavulanic acid and ticarcillin to make Timentin clavulanic acid and amoxicllin to make augmentin
Difference among the beta lactamase inhibitors
Clavulanic acid is the most stable Tazobactam is the most active Sulbactam is somewhere in between
Ticarcillin is not available alone, it is available with what? When do you use this formulation?
Clavulonic acid in the form of Timentin Cystic Fibrosis
A 59 year old man with ACS is admitted to hospital for emergency percutaneous insertion of coronary stent. Which of following drugs might cause unexpected results based on the patients CYP2C19 genotype?
Clopidigrel
A patient has a severe bacterial infection that normally would respond to an oral penicillin or a cephalosporin. However, his chart documents anaphylactoid reactions to both drugs. Given the history, what drug would be preferred for treating the infection, and also poses the last risk of cross-reactivity and an allergic response? A. Clotrimazole B. Gnetamicin C. Metronidazole D. Tetracycline E. Vancomycin
Clotrimazole is an *antifungal* -similar in property to metronidazole, except metronidazole has antibacterial property too. Gentomycin is an *aminoglycoside* -They work on ribosomes, only have significant activities against gram negative organisms. Metronidazole is an *Antiparasite* Tetracycline *Vancomycin is correct* : cover all the gram Positive that PEN would cover.
4-A 45 year old woman presents with pulmonary hypertension. Which of the following target-treatment pairs is most relevant to this patient?
D Endothelin ambriestan
33-In a patient receiving digoxin for congestive heart failure, conditions that may facilitate the appearance of toxicity include
D Hypomagnesia
13- A 55 year old surgeon has developed symmetric early morning stiffness in her hands. She wishes to take a nonsteroidal anti-inflammatory drug to relieve these symptoms. Which drug is an NSAID that is appropriate for chronic therapy of her arthritis?
D Naproxen
18-A 46 year old man consults you regarding his sexual performance issues. A drug that is used in the treatment of male erectile dysfunction and inhibits a phosphodiesterase is
D Sildenafil
13-in considering medical therapy for this patient, which of the following best describes the beneficial action of propanolol in this condition?
D block in beta receptors results in decrease cardiac work
42-the research division of a pharmaceutical corporation has characterized the receptor-blocking actions of 5 new drugs, each of which may have potential therapeutic value. The relative intensities of their blocking actions are shown in following table. Because each of these drugs is lipophilic and can cross the blood brain barrier, they are expected to have CNS effects. Based on data in table, which drug is most likely to exacerbate symptoms of Parkinsons disease?
D drug D
75-You are on the hospital pharmacy committee and revising the formulary. Relative to loratadine, diphenhydramine is more likely to
D have efficacy in prevention of motion sickness
55-The mechanism of action of the hypnotic drug zolpidem is
D increased GABA mediated chloride ion conductance
85-A 24 year old man with a history of partial seizures has been treated with standard anticonvulsants for several years. He is currently taking valproic acid, which is not fully effective, and his neurologist prescribes another drug approved for adjunctive use in partial seizures. Unfortunately the patient develops a toxic epidermal necrolysis. The second drug prescribed is
D lamotrigine
54- A patient taking medications for a psychiatric disorder develops a tremor, thyroid enlargement, edema, and acneiform eruptions on the face. The drug he is taking is most likely to be
D lithium
84-A 45 year old woman suffers from abdominal pain and bloody diarrhea that has been diagnosed as Crohns disease. Which of the following is a first line drug treatment of Crohns disease that acts locally in the gastrointestinal tract to provide an anti inflammatory effect?
D mesalamine
77-A 34 year old woman presented with nervousness, increased perspiration, tachycardia, hand tremors, insomnia, and thinning of the skin. Hyperthyroidism was confirmed. Which of the following is a drug that inhibits the synthesis of thyroid hormone by preventing coupling of idotyrosine molecules?
D methimazole
49-Regarding drugs that relax skeletal muscle, which one of the following statements is accurate?
D muscle relaxation caused by succinylcholine is not reversed by acetylcholinesterase inhibitors
18-A 70 year old woman with mild to moderate hypertension fell 2 years ago during a spell of dizziness and broke her hip. During the last 18 months, her BP has increased. Now she is being treated for BP 160/95. When treating HTN chronically, orthostatic hypotension is greatest with:
D peripheral acting alpha 1 antagonist
88-Which of the following pairs of drug and indication is accurate?
D pramipexole: Parkinsons disease
17-An important difference between nonselective alpha receptor antagonist and and alpha 1 selective antagonists is that alpha 1 selective antagonists
D produce less refulx tachycardia
95-AP is a 46 year old woman who experienced progressive weight loss, tachycardia (110bpm), nervousness, anxiety, and a slight tremor. She has been on propanolol for the last three months and her symptoms are not under control. Which of the following should be next step in the treatment of this patient?
D radioactive iodine
40-A 55 year old patient with a 30 pack year history of smoking and COPD reports frequent episodes of bronchospasm. She has been using an over-the-counter bronchodilator inhaler but complains of palpitations and chest pain when she uses the inhaler. Which of the following would relieve her bronchospasm without causing cardiac stimulation?
D tiotropium
80-JT underwent successful cardiac surgery. At the end of the procedure protamine was administered. Protamine can be used to partially reverse the anticoagulant effect of which of the following?
D unfractionated heparin
Which of the following medications for gastrointestinal problems is contraindicated in pregnancy? A. Calcium carbonate. B. Famotidine. C. Lansoprazole. D. Misoprostol.
D. Misoprostol
Which one of the following statements best describes the actions or effects of esomeprazole and other drugs in its class? A. Cause strong systemic atropine-like (antimuscarinic) effects that limit their use in patients for whom atropine itself is contraindicated B. Inhibit gastric acid secretion by simultaneously and competitively blocking the actions of the agonists histamine, acetylcholine, and gastrin, on their parietal cell membrane receptors C. Neutralize gastric acid faster than any other classes of drugs indicated for peptic ulcer disease or gastroesophageal reflux disease (GERD) D. Profoundly inhibit an ATPase located on parietal cell membranes , thereby inhibiting acid secretion E. Tend to cause bradycardia by antagonizing the positive chronotropic effects of histamine on cardiac H1 receptors
D. Profoundly inhibit an ATPase located on parietal cell membranes , thereby inhibiting acid secretion
9. The set of properties that characterize the effects of a drug on the body is called (A)Distribution (B)Permeation (C)Pharmacodynamics (D)Pharmacokinetics (E)Protonation
Definitions. Pharmacodynamics is the term given to drug actions on the body. The answer is C.
PAE : post antibiotic effect? What is it affected by?
Delay before microorganisms frecover and reenter a log-growth period. Species and drug dependent *Size of inoculum* *Type of growth medium* *bacterial growth phase*
Major differences between the antistaph penicillins?
Dicloxacillin -- oral Naf and Oxacillin are IV only
*Penicillinase-Resistant Penicillins* This drug has the most complete intestinal absorption, PO only
Dixloxacillin
77-A 46 year old man has hypertension of 155/95. His cardiac and kidney function are normal. Valsartan has been suggested as therapy. This drug provides an antihypertensive effect by which of the following actions?
E interfering with the combination of angiotensin II with its receptor
5-A 45 year old man with a duodenal ulcer and lab evidence of H. Pylori infection was treated with omeprazole, clarithromycin, and amoxicillin. Endoscopy revealed rapid healing of the ulcer. Which of the following is the most accurate description of the mechanism of omeprazoles therapeutic action?
E irreversible inactivation of H/K ATPase
1-A patient is admitted to the emergency department with signs and symptoms that could be due to either a muscarinic stimulant or an opioid. Which of the following is a common effect of both muscarinic stimulant drugs and opioids?
E miosis
19-A physician was considering erythromycin for treatment of a 47 year old man with an upper respiratory tract infection. However, the physician noted that the patient was taking simvastatin for treatment of hypercholesterolemia and realized that erythromycin, an inhibitor of cytochrome enzymes, would inhibit the metabolism of simvastatin. The physician opted for a different class of antibiotic to avoid exposing the patient to higher concentrations of simvastatin and a risk of dose dependent toxicity. Which of the following is the primary dose-dependent toxicity of simvastatin?
E muscle pain and weakness, possibly progressing to rhabdomylysis
55-A woman taking haloperidol developed a spectrum of adverse effects that included the amenorrhea-galactorrhea syndrome and extrapyramidal dysfunction. Another, newer, antipsychotic drug was prescribed which however caused weight gain and hyperglycemia due to a diabetogenic action. The drug prescribed was
E olanzapine
19-A 52 year old woman is admitted to the emergency department with a history of drug treatment for several conditions. Her serum electrolytes are found to be as follows: NA 140, CL 100, K 6.5, pH 7.3 This patient has probably been taking
E spironolactodone
58-A 54 year old woman with a recent history of deep vein thrombosis had been stable on warfarin therapy for the last two months. However, her most recent prothrombin time (PT) test revealed a markedly reduced INR. When asked about changes in diet or medication during the last several weeks, the woman said that she had recently begun taking a dietary supplement recommended by a friend. Based on this information, the supplement is most likely to contain which of the following?
E st johns wort
46-A young woman suffering from myoclonic seizures was receiving effective single drug therapy. Because she was planning a pregnancy, her physician switched her to an alternative medication with much less potential for teratogenicity. The original antiseizure drug prescribed for this patient was most likely to have been
E valproic acid
14-A drug that is useful in angina but causes constipation, edema, and increased cardiac size is
E verapamil
83-A 54 year old woman was treated with anticoagulants for deep vein thrombosis. Four days after starting the medication, she noticed pain and redness with black skin under her breasts. The physician diagnosed dermal vascular necrosis resulting from protein C deficiency. Her condition was caused by which one of the following drugs?
E warfarin
A pt has multiple undiagnosed gastric ulcers. Shortly after consuming a large meal and large amounts of alcohol, he experiences significant GI distress. He takes an over-the-counter heartburn remedy. Within a minute or two he develops what he will later describe as a "bad bloated feeling." Several of the ulcers have begun to bleed and he experiences searing pain. The endoscopist remarks that it appears as if the lesions had been literally stretched apart, causing additional tissue damage that led to the hemorrhage. Which drug or product did the patient most likely take? A. An aluminum salt B. A magnesium-containing antacid C. Bismuth subsalicylate D. Ranitidine E. Sodium bicarbonate
E. Sodium bicarbonate HCl + NaHCO3 --- > CO2 gas
Which mechanism explains aspirins inhibition of thromboxane synthesis?
blocks cyclooxygenase
Infections that would benefit from the treatment of synergistic effect b/w PEN and AMG?
Endocarditis!
2. Botulinum toxin is a large protein molecule. Its action on cholinergic transmission depends on an intracellular action within nerve endings. Which one of the following processes is best suited for permeation of very large protein molecules into cells? (A)Aqueous diffusion (B)Aqueous hydrolysis (C)Endocytosis (D)Lipid diffusion (E)Special carrier transport
Endocytosis is an important mechanism for transport of very large molecules across membranes. Aqueous diffusion is not involved in transport across the lipid barrier of cell membranes. Lipid diffusion and special carrier transport are common for smaller molecules. Hydrolysis has nothing to do with the mechanisms of permeation; rather, hydrolysis is one mechanism of drug metabolism. The answer is C.
What is Bactericidal?
Eradicates without host defense mechanisms
· Which statement concerning the proposed mechanism of action of anticonvulsant drugs is inaccurate?
Ethosuximide selectivity blocks potassium ions (K+) channels in thalamic neurons The MOA of phenylsuccinimides such as ethosuximide involves blockage of T-type Ca2 channels in thalamic neurons. Ethosuximide does not block K+ channels, which in any case would likely to result in an increase (rather than a decrease) in neuronal excitability.
Skin falling off in big chunks as a result of toxicity to beta lactam pens?
Exfoliative dermatitis, SJS
Natural Penicillins -Special about them? -Spectrum -Clinical Role -Synergism with? -What to do with someone with an allergy?
Fermentation media used to culture Penicillium PEN G is prototype, greatest activity Susceptible to beta lactamases *Spectrum* Gram +, Spirochetes, anaerobes Bactericidal, but some remittances *clinical role* DoC for syphillis, Strepthroat (upper resp) in those who are not resistant *synergism with* AMG *Allergy?* Give them PEN anyway, just desensitize the shit out of them
Minimum Bacterial Concentration (MBC) requires which of the follows:
First determining the MIC
· H2 antagonists reduce acid secretion stimulated by histamine as well as by gastrin and cholinomimetic agents through two mechanisms:
First, histamine released from ECL cells by gastrin or vagal stimulation is blocked from binding to the parietal cell H2 receptor. Then second, direct stimulation of the parietal cell by gastrin or acetylcholine has a diminished effect on acid secretion in the presence of H2-receptor blockade
Antibiotics work in many different ways. Which of the following antibiotics works by inhibiting DNA synthesis?
Fluoroquinolones like Ciprofloxacin
Where did beta lactams come from?
Fungus (75% roughly of all ABX)
A 65 year old woman has been admitted to the coronary care unit with left ventricular myocardial infarction. She develops acute severe heart failure with marked pulmonary edema, but no evidence of peripheral edema. Which one of the following drugs would be most useful?
Furosemide
· Which of the following drugs promotes release of endogenous insulin?
Glipizide A second generation sulfonylurea that promotes insulin release by closing potassium channel in pancreatic B cells
*Extended Spectrum Pens* Spectrum? Cidal or static? Susceptible to ? Synergisitc with ?
Gram + Gram - anaerobes and Pseudomonas Bactericidal Suscpetible to penicillinases (- inhibitor) Synergistic with AMG
*aminopenicillins* spectrum? Cidal or static? Synergisitic with? Clinical role?
Gram +, Gram -, Anaerobes They are susceptible to penicillinases They are bactericidal Synergistic with AMG Clinical roles : Sinusitis, drug of choice for OM
Ticarcillin is a bit more active against what organisms than Piperacillin is?
Gram positive and some gram negatives
Narrow spectrum penicillin's, both penicillinase-sensitive and penicillinase-resistant have relatively poor activity against gram-negative bacteria. What is the main property or characteristic that explains why gram-negative bacteria do not respond well to penicillin?
Gram-negative bacteria have an outer membrane that serves as a physical barrier to the penicillin binding proteins (PBP's)
Interstitial nephritis as a toxicity of beta lactams. Just give what you remember.....!
Hematuria, albuminuria, pyruia, Elevated SCr, oliguria On biopsy, mononuclear infiltrate -eosinophilia, tubular damage, igG in interstitium REVERSABLE
Distribution and elimination of the natural penicillins?
High concentrations in liver, bile, kidney, semen, joint fluid, lymph, intestine : *does not get into CSF* eliminated via kidney
Serum sickness mediated by ? Assoicated with what?
IgG mediated, associated with > 1 week use
Difference between location of Gram Negative and Gram Positive Beta lactamases?
In GP beta lactamase is outside the peptidoglycan in GN beta lactamase is inside the peptidogylycan in the "periplasmic space"
· Regarding the clinical use of antidepressant drugs, which statement is accurate?
In selecting an appropriate drug in the treatment of depressions, the history of a patients response to specific drugs is a valuable guide
Why do beta lactams have a difficult time trying penetrate organisms that are not Gm(-) or Gm(+)?
Inability to penetrate due to a capsular outer membrane, inner membrane coated with LPS Porin development (gmnegs) Active efflux pumps (gmnegs)
What cause you to suspect local muscle damage associated with penicillin administration?
Increased serum transaminase, LDH
What is Bacteriostatic?
Inhibit growth, requires host defense mechanisms to eradication the infection.
Weaker analgesia, sedation, miosis, decreased intestinal motility, dysphoria, and hallucinations, describes the effects of stimulation on what receptor / receptors?
Kappa
· Drug metabolism in humans usually results in a product that is
LESS LIPID SOLUBLE THAN THE ORIGINAL DRUG
17 year old patient complains that he develops wheezing and shortness of breath whenever he takes aspirin for headache. Increased levels of which of the following may be responsible, in part, in some cases of aspirin hypersensitivity?
Leukotriene LTC4
Ampicillin is the DoC for this bug : it is the cause of meningitis is elderly patients and neonates Pregnant women are not supposed to eat lunch meat because lunch meat contains this
Lysteria or meningites
*Spectrum/Special features : Antistaph Pens* Prototype? Why is it not used in US anymore? Resistant to what? What are the types of bugs its used for? Cidal or static? Clinical role?
Methicillin is the prototype It is not used anymore due to its nephrotoxicity They are resistant to hydrolysis by Staph Penicillinase They are used against Gram + and MSSA They are Bactericidal Clinical is is for Staph, not the MRSA form Less active on other Pen susceptible organisms
What is the MIC?
Minimum concentration you need to visibly reduce the concentration of bacteria
Mr J is 54 years old and currently under care for metastatic bone disease. He has progressed to needing multiple levels of pain management for his severe pain. In consideration of the WHO Pain Progression algorithm of treatment, prescribe appropriate pain management for Mr J:
Morphine plus Cox-2 inhibitor
*Penicillinase-Resistant Penicillins* Has slightly different structure than the others, and this drug is the most active with adequate concentrations in CSF
Nafcillin
Penicillinase-Resistant penicillins (antistaph) Name them
Nafcillin Methacillin Oxacillin
Best coverage for staph ?
Nafcillin, Oxacillin, Dicloxacillin
Staphyloccocus caused meningitis you would use which drug?
Nafcillin, only drug that has adequate coverage in the CSF
You are the lead advanced practice nurse at your pain management practice. You just completed an in-service about treating patients with opioid use disorder. Which best medication did you advise your team to use when treating patients with opioid use disorder, so to avoid the effects of opioids?
Naltrexone
Classes of Penicillins?
Natural Penicillins Aminopenicillins Penicillinase-Resistant Penicillins (anti-staphylococcal) Extended-spectrum penicillins
What are the possible mechanisms of PAE?
Nonlethal bacterial damage Persistance of antimicrobial at site of action -rapid growth when new organisms injected -not caused by persistence of drug in tissue
6. Which of the following statements is most correct regarding the termination of drug action? (A)Drugs must be excreted from the body to terminate their action (B)Metabolism of drugs always increases their water solubility (C)Metabolism of drugs always abolishes their pharmacologic activity (D)Hepatic metabolism and renal excretion are the two most important mechanisms involved (E)Distribution of a drug out of the bloodstream terminates the drug's effects
Note the "trigger" words ("must," "always") in choices A,B, and C, see Appendix IV. The answer is D.
3. A 60-year-old patient with severe cancer pain is given 10 mg of morphine by mouth. The plasma concentration is found to be only 30% of that found after intravenous administration of the same dose. Which of the following terms describes the process by which the amount of active drug in the body is reducedafter administration but before entering the systemic circulation? (A)Excretion (B)First-order elimination (C)First-pass effect (D)Metabolism (E)Pharmacokinetics
One can often determine the relevance of the clinical data by scanning the list of answers, see Appendix IV. In this question, the emphasis is clearly on pharmacokinetic principles. "First-pass effect" is the term given to elimination of a drug before it enters the systemic circulation (ie, on its first pass through the portal circulation and liver). The first-pass effect is usually, but not always, due to metabolism in the gut, the portal blood, or the liver. The answer is C.
Time dependent really means what?
Only active if they are above the MIC
Mechanisms of resistance of beta Lactams?
Outer surface of cytpolasmic membrane, cleave the beta lactam, common with Gram POSITIVE
*Penicillinase-Resistant Penicillins* This drug has variability in intestinal absorption, so it is given IV only
Oxacillin
Acquired resistance to beta lactams?
PBPs that have decreased affinity
Difference between PKvs.PD
PK : What body does to drug (ADME) PD: What drug does to body
Name the natural penicillins
Pen G Pen V (oral)
Form of Pen G that is used for syphillis?
Pen G Benzathine
Penicillin that is more stable with acidic environments, its primary role is oropharyngeal infections
Penicillin V
What is the rational behind separate IV lines? What penicillin should you use if you are not achieving the clinical response you would like to see?
Penicillin may inactivate gentamicin, but not the other way around.
Provide a resistance mechanism that gram negatives have against beta Lactams?
Porin development in the outer membrane
56 year old man has hypertension and an enlarged prostate, which biopsy shows to be benign prostatic hyperplasia. He complains of urinary retention. Which of the following drugs would be the most appropriate initial therapy?
Prazosin
*aminopenicillins* What are they used for mostly?
Primarily you see them for otitis media infections
Piperacillin has better coverage for which organisms compared to ticarcillin?
Pseudomonas
Your patient was diagnosed with pneumonia. Your culture and sensitivities report identified the offending organisms as a gram negative bacilli. You appropriately select an antibiotic capable of clearing which organism
Pseudomonas aeruginosa
1. A 3-year-old is brought to the emergency department hav-ing just ingested a large overdose of diphenhydramine, an antihistaminic drug. Diphenhydramine is a weak base with a pKa of 8.8. It is capable of entering most tissues, including the brain. On physical examination, the heart rate is 100/min,blood pressure 90/50 mm Hg, and respiratory rate 20/min. Which of the following statements about this case of diphen-hydramine overdose is most correct?(A)Urinary excretion would be accelerated by administration of NH4Cl, an acidifying agent (B)Urinary excretion would be accelerated by giving NaHCO3, an alkalinizing agent (C)More of the drug would be ionized at blood pH than at stomach pH (D)Absorption of the drug would be faster from the stomach than from the small intestine (E)Hemodialysis is the only effective therapy
Questions that deal with acid-base (Henderson-Hasselbalch) manipulations are common on examinations. Since absorption involves permeation across lipid membranes, we can in theory treat an overdose by decreasing absorption from the gut and reabsorption from the tubular urine by making the drug less lipid-soluble. Ionization attracts water molecules and decreases lipid solubility. Diphenhydramine is a weak base, which means that it is more ionized when protonated, ie, at acid pH. Choice C suggests that the drug would be more ionized at pH 7.4 than at pH 2.0, which is clearly wrong. Choice D says (in effect) that the more ionized form is absorbed faster, which is incorrect. A and B are opposites because NH4Cl is an acidifying salt and sodium bicarbonate an alkalinizing one. (From the point of view of test strategy, opposites in a list of answers always deserve careful attention.) E is a distracter. Because an acid environment favors ionization of a weak base, we should give NH4Cl. The answer is A. Note that clinical management of overdose involves many other considerations in addition to trapping the drug in urine; manipulation of urine pH may be contraindicated for other reasons.
· Your patient has bacterial sinusitis. You have written the following prescription for your patient: Jane Doe Date - June 9, 2015 Augmentin 875/125mg Dispense #14 SIG: take one tablet by mouth every 12 hours with meals Refills 0 (Zero) Which aspect of this prescription is aimed at addressing the drugs half-life, preventing toxicity and obtaining the greatest benefit and cure from therapy
SIG: take one tablet by mouth every 12 hours with meals
*aminopenicillins* Special features : Side chains alter susceptibility to_____ _____ splits the peptide linkage Joins penicillin to _____________
Side chains alter susceptibility to beta lactamases Amidase splits the peptide linkage Joins penicillin to 6-aminopenicillanic acid
St Johns Wort is a potent inducer of hepatic phase CYP3A4 enzymes. Verapamil and phenytoin are both eliminated from body by metabolism in liver. Verapamil has clearance of 1.5L/min, approximately equal to liver blood flow, whereas phenytoin has clearance of 0.1L/min. Based on this fact which is correct
St Johns Wort will increase clearance of phenytoin
Major gram positive bugs?
Staph Strep Enterococcus
Your patient is diagnosed with hospital-acquired pneumonia. You prepare to treat the patient for gram-positive cocci most likely to have made her ill. The offending organism is most likely __________________to be treated with _____________.
Staphylococcus aureus - Vancomycin
Oxacillin, Nafcillin, Dicloxacillin primarily used for ?
Staphylococcus infections
Which statement/s about opioid pharmacology is/ are most accurate? (Select all that Apply)
Stimulation of the mu receptors results in analgesia, sedation, miosis, respiratory depression, cough suppression, euphoria, and decreased GI motility Opioids modulate nociception in the terminals of afferent nerves in the central nervous system, peripheral nervous system, and GI tract
Walter is a 30-year-old graduate student working on his adult nurse practitioner training. He works currently as an ER/trauma nurse. In is free time he loves the sport of wrestling. He presents to you for evaluation and treatment of a large spontaneous abscess that developed on his right knee. He is confident this abscess is secondary to acquiring an abrasion while wrestling; he insists this is a MRSA abscess. He asks that you explain what causes MRSA. Which is the most accurate explanation of the cause of MRSA?
Structural changes in target penicillin-binding proteins (PBP's) are responsible for methicillin resistance
Intrinsic resitance to beta Lactams?
Structural difference in PBP
Sulbactam is combined with what drug to make what?
Sulbactam is paired with ampicillin to make unasyn
· When initiating T4 therapy with an elderly patient with longstanding hypothyroidism, which symptom most accurately suggest you progressed too rapidly in dosing therapy?
Symptoms of over stimulation of the heart
Tazobactam is paired generally with what?
Tazobactam generally is paired with piperacillin to make zosyn
*aminopenicillins* Special features : Stable in _____ meaning they are intact after__ *administration* Ampicillin is given_____ Amoxicillin is given______ Amoxicillin is more rapidly________, and incidence of ________is less
The aminopenicillins are stable in acid, meaning they are intact after PO administration. Ampicillin is given PO or IV Amoxicillin is given PO only Amoxicillin is more rapidly absorbed from GI tract is incidence of diarrhea is less.
Because of genetic modifications and adaptation, you are addressing the risks of antibiotic use. Which statement most accurately addresses the result of a survival advantage?
The bacterial clone ultimately becomes the dominate bacterial flora.
8. The pharmacokinetic process that distinguishes the elimination of ethanol and high doses of phenytoin and aspirin from the elimination of most other drugs is called (A)Distribution (B)Excretion (C)First-pass effect (D)First-order elimination (E)Zero-order elimination
The excretion of most drugs follows first-order kinetics. However, ethanol and, in higher doses, aspirin and phenytoin follow zero-order kinetics; that is, their elimination rates are constant regardless of blood concentration. The answer is E.
Which statement most accurately describes the minimum inhibitory concentration (MIC)?
The lowest concentration of antibiotic that prevents active bacterial growth constitutes the MIC.
Some bacteria contain the enzyme β-lactamase. Which statements are true about β-lactamase? (Select All That Apply)
The most common mode of drug resistance is plasmid transfer of the genetic code for the β-lactamase enzyme There are β-lactamase for both the penicillin's and cephalosporin's
You are lecturing a group of stellar Rutgers Doctors of Nursing practice students, about antibiotic selection, and the need to first understand the microbiology of organisms. You ask the students which principle, in part, guides their selection of antibiotics.
The principle of colonization that addresses the harmless presence of organism's living in harmony
What are the three main components of beta lactams
Thiazolidine ring (house) Beta lactam ring (garage) side chain
7. Which statement about the distribution of drugs to specific tissues is most correct? (A)Distribution to an organ is independent of blood flow (B)Distribution is independent of the solubility of the drug in that tissue (C)Distribution depends on the unbound drug concentration gradient between blood and the tissue (D)Distribution is increased for drugs that are strongly bound to plasma proteins (E)Distribution has no effect on the half-life of the drug
This is a straightforward question of pharmacokinetic distribution concepts. From the list of determinants of drug distribution given on page 6, choice C is correct.
Extended Spectrum Penicillins?
Ticarcillin Piperacillin
*Extended Spectrum Pens* Differnce between the two ?
Ticarcillin (carboxyl pen) Piperacillin is better because more coverage
Toxicity of Beta lactams -Act as ____ after reaction with proteins -Large % of ____reactions to breakdown product ______detectable in virtually all who have received the drug Recent exposure can be ____________
Toxicity of beta lactams Act as happens after reactions with proteins Large % of IgE reactions to breakdown product Anti-Pen abs detectable in virtually all who have received the drug Recent exposure can be skin sensitizing
*Extended Spectrum Pens* Ureidopenicillins Drugs we think of? Superior activity to what? With ________ this drug has broadest activity of all penicillins!
Ureidopenicillins we think of Mexlocillin, piperacillin Superior activity against Pseudomonas, with tazobactam, piperacillin has broadest activity of all penicillins.
General statement about half lives, dosing, and toxicities of the anti-staph pens?
Very short, must be administered every 4 to 6 hours
50 year old man has history of frequent episodes of renal colic with calcium-containing renal stones. A careful workup indicates that he has a defect in proximal tubular calcium reabsorption, which results in high concentrations of calcium salts in the tubular urine. The most useful diuretic agent in the treatment of recurrent calcium stones is
chlorthalidone
What drug(s) is/are classifed as antidiarrheal? (Select all that apply.) a. Adsorbents b. Anticholinergics c. Opioids d. Proton pump inhibitors e. Selective serotonin reuptake inhibitors
a. Adsorbents b. Anticholinergics c. Opioids
Of the following groups of medications, which can be used as antiemetics? (Select all that apply.) a. Anticholinergics b. Antihistamines c. Cannabinoids d. Opioids e. Phenothiazines
a. Anticholinergics b. Antihistamines c. Cannabinoids e. Phenothiazines
Which statement best describes an action or property of the COX-2 inhibitor, celecoxib, when compared to non-selective cyclooxygenase inhibitors? a. Associated with a lower risk of gastric or duodenal ulcer. b. Cures arthritis, rather than just give symptom relief. c. Effectively inhibits uric acid synthesis. d. Has a lower risk of adverse or fatal cardiac events. e. Has significantly faster onset of action.
a. Associated with a lower risk of gastric or duodenal ulcer.
The patient has been prescribed promethazine (Phenergan) 25 mg PO for nausea that accompanies her migraines. What side effect(s) should be included in the patient teaching? (Select all that apply.) a. Blurred vision b. Diarrhea c. Drowsiness d. Dry mouth e. Hypotension
a. Blurred vision c. Drowsiness d. Dry mouth e. Hypotension
Which area(s) in the brain cause(s) vomiting when stimulated? (Select all that apply.) a. Chemoreceptor trigger zone b. Nausea center c. Medulla d. Vertigo center e. Vomiting center
a. Chemoreceptor trigger zone e. Vomiting center
The patient is receiving diphenoxylate with atropine (Lomotil) for treatment of diarrhea. What side effect(s) might the PA expect to see during treatment? (Select all that apply.) a. Dilated pupils b. Drowsiness c. Hypertension d. Hypoglycemia e. Urinary retention
a. Dilated pupils b. Drowsiness e. Urinary retention
A patient receives a drug that inhibits the lipoxygenase pathway of arachidonic acid metabolism. What naturally occurring metabolite(s) will be produced in lesser amounts in response to this drug? a . Leukotrienes b. Prostacyclin (PGI2) c. Prostaglandins d. Thromboxanes e. Uric acid
a. Leukotrienes
A patient presents in the ED with an overdose of a drug. The PA knows what the drug is and orders multiple doses of N-acetylcysteine. Upon which drug did the patient overdose? a. acetaminophen b. aspirin c. heroin d. methotrexate e. oxycodone
a. acetaminophen
The patient has severe nausea and vomiting and has been prescribed promethazine 25 mg PO q4-6h. The patient asks how the medication works. What is the PA's best response? a. "It stimulates the dopamine receptors in the brain associated with vomiting." b. "It blocks the histamine receptor sites and inhibits the CTZ." c. "It blocks the acetylcholine receptors associated with vomiting." d. "It prohibits the muscle contraction in the abdominal wall, preventing vomiting."
b. "It blocks the histamine receptor sites and inhibits the CTZ."
With regards to the gastrointestinal system, what do opiates do? a. Absorb toxic materials b. Decrease intestinal motility c. Gently stimulate peristalsis d. Stimulate the CTZ
b. Decrease intestinal motility
70 year old retired businessman with history of chronic heart failure has been taking digoxin and furosemide. He is now admitted with a history of vomiting, acute decompensated heart failure, and metabolic derangements. He has marked peripheral edema and metabolic alkalosis. Which of the following drugs is most appropriate for the treatment of his edema?
acetazolamide
Which of the following is primary neurotransmitter agent normally released in the SA node of heart in response to a blood pressure increase?
acetylcholine
Your 75 year old patient with multiple comorbidities is to receive beta blocking drug. For which of following conditions are beta blocking drugs not indicated?
acute arrhythmias in surgery atherosclerotic angina pectoris (angina effort) chronic heart failure hypertension -->hypoglycemia in diabetes
Clinical studies have shown that adrenoreceptor blockers have many useful effects in patients. However, a number of drug toxicities have been documented. Adverse effects limit the use of adrenoreceptor blockers include which of following?
acute heart failure exacerbation from beta blockers
73 year old man with history of a recent change in his treatment for moderately severe hypertension is brought to emergency department because of a fall at home. Which of the following drug groups is most likely to cause postural hypotension and thus increased risk for falls?
alpha 1 selective receptor blockers
which of the following drugs may inhibit hepatic microsomal P450 responsible for warfarin metabolism?
amiodarone
inadequate response to other drugs is about to receive digoxin for chronic heart failure. NSR HR 88 and BP 135/85 Which of the following is the best documented mechanism of beneficial action of cardiac glycosides?
an increase in systolic cytoplasmic calcium levels
concerned with antibiotics What does time-dependent imply?
antimicrobial activity directly related to the : *time spent above the MIC*
Concerned with ABX What does AUC dependent imply?
antimicrobial activity related to : *Total exposure of the antibiotics versus MIC*
Pasturella in dog bites, people bites...which treatment...
augmentin
Suicide inhibitor
beta lactam inhibitors
Comparison with prazosin and atenolol shows that
both increase sympathetic outflow from CNS
Misoprostol, an analog of PGE1, is sometimes used adjunctively to stimulate gastric mucous production and help reduce the incidence of gastric ulcers associated with long-term or high-dose NSAID therapy for arthritis. What is the other main use for this lipid-derived autacoid? a. Closure of a PDA in newborns. b. Contraception in women who should not receive estrogens or progestins. c. Induction of abortion. d. Prophylaxis of asthma in lieu of a corticosteroid. e. Suppression of uterine contraction in women with premature labor.
c. Induction of abortion.
What is the most likely mechanism by which infliximab suppresses the signs, symptoms, or underlying pathophysiology of Crohn's disease? a. Inhibits eicosanoid synthesis by inhibiting phospholipase A2 b. Inhibits leukocyte migration by blocking microtubular formation c. Neutralizes circulating TNF-alpha d. Selectively and effectively inhibits COX-2 e. Stimulates collagen and mucopolysaccharide in the intestine.
c. Neutralizes circulating TNF-alpha
Which of the following has been shown to prolong life in patients with chronic congestive failure in spite of having a negative inotropic effect on cardiac contractility?
carvediolol
Which of the following factors is likely to increase the duration of action of a drug that is metabolized by CYP34A in the liver?
chronic therapy with amiodarone
Which of the following is an important effect of chronic therapy with loop diuretics?
decreased urinary excretion of calcium elevation of BP elevation of pulmonary vascular pressure teratogenic action in pregnancy -->metabolic alkalosis
which of the following is characteristic of nifedipine treatment in patients with essential hypertension?
decreases calcium influx into smooth muscle
With which drug should concomitant use of allopurinol be avoided/cautioned? a. Omeprazole b. Cisplatin c. Prednisone d. Sulfasalazine e. Azathioprine
e. Azathioprine Allopurinol interferes with the metabolism of azathioprine, increasing levels of mercaptopurine. Dosage of azathioprine should be reduced in patients receiving allopurinol.
While reviewing charts in a general medicine clinic you see that a patient, 55-year-old and with no history of cancer at all, is taking methotrexate (MTX). What is the most likely condition for which this "anticancer drug" is being given? a . Severe nausea and vomiting b. Severe, refractory diarrhea c. Peptic ulcer disease with H. pylori d. Myasthenia gravis e. Rheumatoid arthritis or psoriasis
e. Rheumatoid arthritis or psoriasis
You are considering the use of a pure agonist, specifically an opioid analgesic, for a patient newly diagnosed with early bone metastasis. You have several selections because you know different opioid agonists can provide equivalent analgesia, if given in appropriate doses. This is referred to as:
equianalgesic doses that may have differing side effect profiles
68 year old man with history of chronic heart failure goes on vacation and abandons his low salt diet. Three days later, he develops severe shortness of breath and is admitted to the local hospital emergency department with significant pulmonary edema. The first line drug of choice in most cases of acute decompensation in patients with chronic heart failure is
furosemide
70 year old woman is admitted to the emergency department because of a "fainting spell" at home. She appears to have suffered no trauma from her fall, but her BP 120/60 when lying down and 60/20 when she sits up. Neurologic exam and an ECG are WNL when she is lying down. Questioning reveals that she has recently started taking "water pills" (diuretics) for a heart condition. Which of the following drugs is most likely the cause of her fainting spell?
furosemide
A 62 year old man with advanced prostate cancer is admitted to the emergency department with mental obtundation. An electrolyte panel shows a serum calcium of 16.5. Which of the following therapies would be most useful in management of severe hypercalcemia?
furosemide plus saline solution
Vicodin (hydrocodone 7.5 / acetaminophen 750). Sig: 1 tablet by mouth every 4-6 hours as needed for pain. In consideration of the WHO Pain ladder, this prescription is for a patient who:
has moderate pain
Which of the following is a significant unwanted effect of the drug named?
heart failure and hydralazine hemolytic anemia and atenolol hypokalemia and aliskiren lupus like syndrome with HCTZ -->constipation with verapamil
A patient with a positive fecal blood test is referred to the gastroenterology clinic. He is apparently taking large amounts of an unidentified drug that inhibits platelet activity. Which of the following is taken orally and directly and reversibly inhibits platelet cyclooxygenase?
ibuprofen
Significant number of patients started on ACE inhibitor therapy for hypertension are intolerant and must be switched to a different class of drug. What is most common manifestation of this intolerance?
incessant cough
a patient with hypertension and angina is referred for treatment. Metoprolol and verapamil are among the drugs considered. Both metoprolol and verapamil are associated with which of the following?
increased PR interval
A patient who has been taking digoxin for several years for atrial fibrillation and chronic heart failure is about to receive atropine for another condition. A common effect of digoxin (at therapeutic drug levels) that can be almost entirely blocked by atropine is
increased PR on ECG
inadequate response to other drugs is about to receive digoxin for chronic heart failure. NSR HR 88 and BP 135/85 After your patient has been receiving digoxin for 3 weeks, he presents to the emergency department with an arrhythmia. Which one of the following is most likely to contribute the arrhythmogenic effect of digoxin?
increased intracellular calcium
54 year old airline pilot is admitted to the emergency department with chest pain and rapid heart rhythm. The ECG shows inferior myocardial infarction and ventricular tachycardia. Amiodarone is ordered. Amiodarone
increases action potential duration
Why is pseudomembranous colitis a toxicity of penicillin administration?
it eliminates part of the GI tract flora (diarrhea) overgrowth and production of toxin by C. Difficile
Why would you not want to give Pen V with someone with an arrhythmia?
it is formulated with potassium salts, and this could upset the electrolytes and further complicate the arrhythmia
32 year old woman with hypertension wishes to become pregnant. Her physician informs her that she will have to switch to another antihypertensive drug. Which of following drugs is absolutely contraindicated in pregnancy?
losartan
Patient admitted to the emergency department with severe tacycardia after a drug overdose. His family reports that he has been depressed about his hypertension. Which of the following drugs increases heart rate in dose-dependent manner?
minoxidil
Which of the following is leukotriene receptor blocker?
montelukast
· Mary has newly diagnosed type 2 diabetes. Your goal of therapy is to do the following: Decreases hepatic glucose production; Increases insulin sensitivity; first line for most type 2 diabetes patients and contraindicated for Type 1 DM. You are prescribing which?
o Biguanides
· Complete this Sentence: a partial agonist's inability to cause a maximal pharmacologic response, even when in high concentrations sufficient to saturate binding to all receptors, happens because... (Select all that Apply) Group of answer choices
o ...partial agonist competitively inhibit the responses produced by full agonist o ...partial agonists produce a lower response, at full receptor occupancy, different from full agonist
· Despite the pending lawsuit, you continue treating your patient John Iwantalawyer, age 43 years, for heterozygous familial hypercholesterolemia. Typically a very pleasant man until recently, he is still mad that you caused his gout exacerbation. You again review his chart and revealed his serum concentration of total cholesterol and LDL are markedly elevated; his HDL, VLDL, and triglycerides are normal or just slightly elevated. You have no alternative but to counsel him on lifestyle and dietary changes, and then you added atorvastatin. Reluctantly, he agrees to start this new prescription of atorvastatin, along with strict quarterly surveillance and routine serology. Before agreeing however, he asked why this is necessary, and what you are specifically watching in his lab work. During his treatment with atorvastatin, you know it is important to routinely monitor serum concentrations of which of the following?
o Alanine & aspartate aminotransferase
· Compared with other antidepressant drugs, mirtazapine has the distinctive ability to act as an antagonist of:
o Alpha2 adrenoceptors
· Johnny Clots is 65 years of age and brought to the ER 30 minutes after experiences the onset of left-sided weakness and aphasia. Imaging studies on arrival ruled out a cerebral hemorrhage. You are the NP on duty today when Johnny Clot arrives. You order the nursing staff to administer which drug to improve Johnny's clinical outcome?
o Alteplase
· You admit Johnny Clots to CCU. Two days after admission and administration of the drug you selected in the previous question, Johnny prepares to go home. You write the discharge instructions that include a prescription for prevention of recurrence of his disease. You know he will need this drug indefinitely. You would write a prescription for which drug?
o Aspirin
· You are teaching your colleagues about various pharmacokinetic principles. You are describing the principle that addresses a fraction of unchanged drug that must reach the systemic circulation, no matter the route, so that the drug can have a therapeutic benefit. This describes which pharmacokinetic principle:
o BIOAVAILABILITY
· Thomas, age 69 has Parkinson's disease along with prostatic hypertrophy and past medical history of obstructive gastrointestinal disease. As the nurse practitioner caring for him you know you must use great caution with this drug or drugs in the same class.
o Benztropine
John has generalized anxiety disorder. He is a 24 year old computer programer who has a nervous disposition. He startles easily. He exaggerates most of his worries, and he worries constantly about inconsequential matters. When he worries the most, he also complains of stomach cramping. He has no history of drug or alcohol abuse. Assuming the symptoms experienced by John are those indicative of generalized anxiety disorder (GAD), the most appropriate drug that you intend to prescribe is which?
o Buspirone ( Buspar) Buspar, and other strong longer acting benzodiazepines are effective in the management of GAD
· Of the following, which statement most accurately identifies some aspect of the MOA of benralizumab (Fasenra)?
o By binding to the human interleukin-5 receptor (IL-5R) Fasenra leads to apoptysis of eosinophils through antibody-dependent cell-mediated cytotoxity.
· You are teaching your classmates about congestive heart failure pharmacology. You accurately teach them by relaying that the drugs that have been found to be least helpful in heart failure are the
o Calcium channel blockers
· A 32 year old woman with hypertension wishes to become pregnant. As her nurse practitioner you advise her she will have to switch to another antihypertensive drug. Which of the following would you not prescribe since it is an absolute contraindication?
o Captopril
· You are presenting Jane to your colleagues in Grand Rounds. Jane has a complex cardiac history. She has cycled through many drugs over the last 10 years, and had had several near death experiences due to cardiac health. In her case, you argue that her prolonged life while living with chronic congestive heart failure was most likely attributed to which drug, even though the drug is known to have a negative inotropic effect on cardiac contractility?
o Carvedilol
· Which of the following gastrointestinal drugs interferes the most with CYP450 enzyme and thus leads to many drug-drug interaction?
o Cimetidine
· One month later Jonny Clot returns with another illness unrelated to his cardiac history. You discover he is unable to tolerate the drug your discharged him on (In Question Number #2) even though he should have been on this indefinitely and you need to make an important change. You ultimately end up changing his drug to clopidogrel because when you compared clopidogrel to ticlopidine, you accurately learned that:
o Clopidogrel is less likely to cause neutropenia
· This MOA: Anti-inflammatory; Reduces airway hyperresponsiveness; Inhibits inflammatory cell migration and activation; Blocks late phase reaction to allergen - describes which respiratory drug?
o Corticosteroids
· Which of the following is characteristic of captopril treatment in patients with essential hypertension?
o Decreases angiotensin II concentration in the blood
· Joe Buff is a 58 year old entrepreneur and owns a well-established exotic bird store. After hurricane like weather and lost power, he suffered the death of many tropical parrots. He is mentally distraught and devastated at the loss of life as well as the financial investment. He was bought to the ER by ambulance about 2 hours after beginning to experience severe chest pain. He has a history of poorly controlled mild hypertension along with hyperlipidemia. He has no history of smoking. In the ER his EKG confirms the diagnosis of myocardial infarction. The medical and cardiac nurse practitioner team has decided to attempt to open his occluded artery.There has been a change of plans in Joe Buff's care. It was decided that Joe Buff needs to undergo a PCA and placement of a stent in the coronary blood vessel. He will be given eptifibatide. Which of the following most accurately describes the MOA of eptifibatide anti-clotting action?
o Eptifibatide causes reversible inhibition of glycoprotein IIb/IIIa receptors
· Which drug administered as an inactive prodrug accumulates in parietal cell canaculi and undergoes conversion to a derivative that irreversibly inhibits the proton pump?
o Esomeprazole
· Which of the following terms describes the process by which the amount of active drug in the body is reduced after the administration but before entering the systemic circulation?
o FIRST PASS EFFECT
· Your patient has an 18-year history of asthma. Select an appropriate Inhaled Corticosteroids (ICS) prescription.
o Fluticasone (Flovent) HFA/MDI 220 mcg/puff. Sig: 2-4 puffs two times per day. Disp #1, Refill 3.
· The first line drug of choice in most cases of heart failure is:
o Furosemide
•Which of the following drugs is most likely to cause hypoglycemia when used as monotherapy in the treatment of type 2 diabetes?
o Glyburide The insulin secretagogues, including the sulfonylurea glyburide can cause hypoglycemia as a result of their ability to increase serum insulin levels. The biguanides, thiazolidinediones, and alpha glucosidase inhibitors are euglycemics that are unlikely to cause hypoglycemia
· Joe Buff is a 58 year old entrepreneur and owns a well-established exotic bird store. After hurricane like weather and lost power, he suffered the death of many tropical parrots. He is mentally distraught and devastated at the loss of life as well as the financial investment. He was bought to the ER by ambulance about 2 hours after beginning to experience severe chest pain. He has a history of poorly controlled mild hypertension along with hyperlipidemia. He has no history of smoking. In the ER his EKG confirms the diagnosis of myocardial infarction. The medical and cardiac nurse practitioner team has decided to attempt to open his occluded artery. If a fibrinolytic drug is used for treatment of Joe Buff's acute MI, which of the following adverse drug effects is the most likely to happen?
o Hemorrhagic stroke
· Your patient has hypertension and angina who also has a known history of coronary artery disease. What prescription did you write as the first choice of therapy for treating this HTN and angina?
o Hydrochlorothiazide 25mg. Sig: one tablet by mouth each day. Disp #30. Refills 1 (one)
· The pharmacokinetic principle that address either passive diffusion or facilitated transport with drug vehicles?
o INCORRECT: DISTRIBUTION Permeation Ionization Absorption Distribution
· Which of the following is a sign or symptom that would be expected to occur in the event of chronic overdose with exogenous T4?
o Weight loss
· Which description most accurately details what needs to be understood by you as the prescriber about the cytochrome P450 hemoprotein?
o INCORRECT: In considering poly-pharmacy, and drug-drug interactions, CYP450 serves as a terminal oxidase that is unaffected by molecular oxygen, flavoprotein, or high lipid solubility -In considering poly-pharmacy, and drug-drug interactions, there are minimal concerns about P450 provided the given drugs are metabolized in the kidneys - In considering poly-pharmacy, and drug-drug interactions, on repeated administration of some substrate drugs this can either enhance the rate of synthesis, or reduce the drugs degradation -In considering poly-pharmacy, and drug-drug interactions, CYP450 serves as a terminal oxidase that is unaffected by molecular oxygen, flavoprotein, or high lipid solubility -In considering poly-pharmacy, and drug-drug interactions, metabolism of drugs and other foreign chemicals will never have an innocuous biochemical event provided the drug is metabolized in the kidney and is not nephrotoxic.
· Which of the following best describes the mechanism of action of benzodiazepines?
o Increase the frequency of opening of chloride ion channels that are coupled to GABAA receptors
· Which of the following is an important effect of insulin?
o Increased glucose transport into the cells Insulin lowers serum glucose concentration in part by driving glucose into the cells, especially into muscle cells.
· Mr. Green is a 65-year old former smoker with cardiac disease and severe COPD associated with frequent episodes of bronchospasm. Which of the following is/ are bronchodilator/s useful in COPD and least likely to cause cardiac arrhythmia?
o Ipratropium
· You are treating a patient with unfractionated heparin who develops severe thrombocytopenia but they still require continued parenteral anticoagulation. You know you would most likely change your prescription to which drug?
o Lepirudin
· John, a 45-year-old healthy male, was attempting to rescue a feral cat that was born in the woods behind his home. John suffered a cat bite to the fifth digit -right hand, and now has osteomyelitis of that finger. As the prescriber, you know to sufficiently treat osteomyelitis this requires 6 weeks of IV vancomycin to penetrate to the bone to sufficiently cure osteomyelitis. In consideration of your BID dosing for cure, you are actually addressing
o Maintenance dose
· Jane has hepatitis C, and hyperthyroidism. She is past her child bearing age, and pregnancy is not a concern. Your preferred antithyroid drug to treat Jane will be which?
o Methimazole Except in pregnancy and thyroid storm methimazole is preferred over propylthiouraciol because of lower risks of severe liver insult
· It is Friday, and your last patient for the day before you leave on a cruise to Alaska, just arrived. Your staff neglected to schedule him as a new patient and they have only allowed a 15 minute spot for him thinking he's a follow up patient that you know well. Instead, you have no history on Mr. Ijust Ruinedyourday, and you have no idea about Mr. Ruinedyourday. As it turns out, Mr. Ruinedyourday is already cranky because you are running late. Because you are a stellar, Rutgers trained NP who can handle anything and you are never rattled - you accept the challenge. Here is what you learned. Mr. Ruinedyourday is a 56 year old man with a 40 year history of smoking, and lives with type 2 diabetes. He recently attended a Rutgers Health fair and learned he is also living with hypertension. You began treatment. Two weeks after starting your treatment plan, he returned with exacerbation of bronchial asthma. Which prescription did you prescribe that had the potential to cause the development of bronchial asthma while also inhibiting gluconeogenesis?
o Metoprolol succinate 25mg. Sig: one tablet by mouth daily. Disp #30. Refills 0 (zero)
· Which of the following drugs is most suitable for management of tremors in a patient with pulmonary disease?
o Metroprolol This is especially important where pulmonary disease contraindicates use of propranolol.
· The MOA of leukotriene modifiers is multifactorial. Specifically, the goals are to block the actions of leukotrienes, decrease the migration of eosinophils, the production of mucus, and bronchoconstriction. Which of the following prescriptions is a correctly prescribed leukotriene modifier for the prevention of exercise-induced bronchoconstriction?
o Montelukast (Singulair®) 10mg. Sig: 1 tablet by mouth daily 2 hours before exercise. Disp #15, Refills 0
· You are treating an age 58 year old man with a history of hyperlipidemia. He has striking hypertriglyceridemia, elevated VLDL cholesterol, and depressed HDL cholesterol. After only 6 months of treatment, his triglyceride and VLDL cholesterol dropped dramatically, and his HDL cholesterol had doubled. Which one drug can you attribute these dramatic changes, particularly the large increase in HDL?
o Niacin
· You are treating your patient John Iwantalawyer, age 43 years, for heterozygous familial hypercholesterolemia. Typically a very pleasant man until recently, he is now mad as all get out. You review his chart and revealed his serum concentration of total cholesterol and LDL are markedly elevated; his HDL, VLDL, and triglycerides are normal or just slightly elevated. You also uncovered his familial history of his mother and brother both died of myocardial infarction before their age of 50. He admits to chest pain when walking up steps "sometimes" and has a diagnosis of angina. He smokes 1 pack of cigarettes per week which he has done for at least 20 years; he is overweight and drinks alcohol most evenings. He's angry today because you overlooked his diagnosis of gout, and sternly said to you "you messed this up big time - both my big toes look like apples on fire, and the pain is all your fault" What recent prescription did you start him on that explains his anger that he has directed at you?
o Niacin
· You have just met your new patient Greg, and diagnosed him with congestive heart failure (CHF). He has multiple comorbidities. In planning your treatment for CHF, you know it is necessary to discontinue which of this medications?
o Nifedipine (Procardia XL) for long term management of chronic stable angina
· When considering this MOA and description- Binds to circulating IgE that leads to the release of inflammation mediators from mast cells. Its indication: adjunct therapy in patients with allergies and severe asthma. Its administration: Subcutaneous injection every 2-4 weeks. You correctly identify which drug?
o Omalizumab (Xolair®)
· Which of the following statement/s about Proton Pump inhibitors is/are true? Option 1: Proton pump inhibitors inhibit both fasting and meal stimulated secretions because they block the final pathway of acid secretion. Option 2: Proton pump inhibitors are administered as inactive prodrugs. Option 3: Proton pump inhibitors have a completely different MOA when compared to H2 antagonist, and a different MOA between each other. Option 4: Proton pump inhibitors have minimal cytochrome P450 interferences. Option 5: Some proton pump inhibitors are sodium salts
o Options 1, 2, 3 and 5 are true
· When initiating T4 therapy for an elderly patient with long-standing hypothyroidism, it is important to begin with small doses to avoid which of the following?
o Over stimulation of the heart patients with longstanding hypothyroidism, especially the elderly, are highly sensitive to the stimulatory effects of t4 on cardiac functions. Administration of even regular doses can cause over stimulation of the heart and cardiac collapse.
· The set of properties that characterize the effects of a drug on the body is called what?
o PHARMACODYNAMICS
· Mr. S is a 45 years old patient you have cared for over many years. He is generally a very health man, despite having poorly controlled psoriasis. His PMH includes only psoriasis. For most of his adult life, because of his plaque psoriasis he often tells you "I look disfigured." He is often depressed because of the plaque and will do anything to improve his skin. He comes to you to ask your opinion about enrolling in a small drug study for people living with psoriasis. He advised you that as part of the trial, his understanding is that he would help in the efficacy of a new psoriasis drug. To him he also understood this to mean he is one of two hundred people with psoriasis whom are helping to prove the worth of the drug. You know that this describes which phase of drug development.
o PHASE 2
· You are discussing multiple options to treat Mr. Breath's gastrointestinal problems. In doing so you discuss adding a proton pump inhibitor. Which prescription is an accurate prescription for a proton pump inhibitor that you are considering for Mr. Breath?
o Pantoprazole (Protonix®) 40mg. Sig: one tablet PO Daily, Dispense #30
· The following adverse drug reactions: dark stools, black tongue, decreased platelet aggregation- are most likely secondary to the use of which drug?
o Pepto Bismol®
· A 65-year old man with multinodular goiter is scheduled for a near-total thyroidectomy. Which of the following drugs will be administered for 10- 14 days before surgery to reduce the vascularity of his thyroid gland?
o Potassium iodine
· This MOA: Inhibit hydrogen potassium pump in parietal cell leading to decreased acid secretion- describes which drug?
o Rabeprazole (Aciphex®)
· On your first day as a new doctoraly prepared nurse practitioner you encounter a patient who is taking a proton pump inhibitor (PPI), plus bismuth, metronidazole, and tetracycline. What is the most likely condition for which this drug combination is being used?
o Refractory or recurrent, and severe, gastric ulcers secondary to Helicobacter pylori
· Which statements concerning the barbiturates is accurate?
o Respiratory depression caused by barbiturates overdosage can be reversed by flumazemil. ?????
· Joe Buff is a 58 year old entrepreneur and owns a well-established exotic bird store. After hurricane like weather and lost power, he suffered the death of many tropical parrots. He is mentally distraught and devastated at the loss of life as well as the financial investment. He was bought to the ER by ambulance about 2 hours after beginning to experience severe chest pain. He has a history of poorly controlled mild hypertension along with hyperlipidemia. He has no history of smoking. In the ER his EKG confirms the diagnosis of myocardial infarction. The medical and cardiac nurse practitioner team has decided to attempt to open his occluded artery.Which of the following drugs would you as the ordering NP be using to accelerate the conversion of plasminogen to plasmin?
o Reteplase
· Which of the following drugs is protective against neurotoxicity of MPTP (1-methyl-4-phenyl-1,2,3,6-tetrahydropyridine), a chemical known to cause the destruction of dopaminergic neurons in the nigrostriatal tract?
o Selegiline
· An 86-year-old woman, otherwise healthy for her age, has difficulty sleeping. Triazolam is prescribed for her at half the conventional adult dose. Which statement about the use of triazolam in this elderly patient is accurate?
o She many experience amnesia, especially if she drinks alcohol beverages.
· When formulating your pharmaceutical plan to treat your patient with GI problems, at the cellular level, you consider the MOA of multiple GI drugs for ulcerative disease. Which statement did you mention in Grand Rounds that is true regarding cell production and the MOA of various drugs to protect the gut?
o Some drugs work to counter the overproductions of gastric acid and intrinsic factor from the parietal cells.
· Because of allergies, your patient who happens to be > 12 years of age, has persistent asthma, and needs to use a high-dosed inhaled corticosteroid (ICS) plus a Long-acting β2 Agonists (LABA's) plus Omalizumab (Xolair®). The utility of multiple drugs indicates this patient has reached which level in the step-wise approach to the management of asthma?
o Step 5
· The range between the minimal toxic dose of a drug and the minimum therapeutic dose is called:
o THERAPEUTIC WINDOW
· For your geriatric patients, you know that age related changes require constant observation of both renal and liver function. Which of the following is a true statement?
o The decline in creatinine clearance occurs in about 2/3's of the geriatric population, and is a late indicator of true kidney function because the production of creatinine is also reduced as muscle mass declines with aging which results in a marked prolongation of the half-life of many drugs
You are caring for a patient in the emergency room admitted for severe bradycardia after a suicide attempt by drug overdose. His family reports he has been depressed since his new diagnosis of hypertension and the side effects of his medication. Which of the following dose-dependent drugs might he have been on if he presents with a slowed heart rate?
o Verapamil
· The amount of a drug in the body compared to the concentration of a drug in blood or plasma, describes what aspect of pharmacokinetics
o Volume of distribution
· In selecting my gastrointestinal drugs for my patient newly diagnosed GERD, as the prescriber I must recall that cells called enterochromaffin-like (ECL) cells have receptors for gastrin and acetylcholine, which stimulate _____________________?
o histamine release.
Mr Rambo is a Hollywood stunt actor. He has significant degenerative disk disease, and multiple disk herniations after years of work. You work in a Pain Management subspecialty practice, and know Mr. Rambo well. You believe he has plateaued in the benefits of using his current drugs. You decide it is time to switch from one opioid to another. Your plan includes:
reducing the total daily dose of the new agent by approximately 25% because there is incomplete cross-tolerance between opioids
· In the elderly, pharmacodynamic changes are a result of changes in the interaction of many drugs with their receptors. While there are many age related concerns, the most concerning family of drugs that the elderly especially have an altered homeostatic response to are?
sedative hypnotics and analgesics drugs
Your 30 year old patient has moderately severe new onset asthma and you prescribe a highly selective beta 2 agonist inhaler to be used when needed. In considering the possible drug effects in this patient, you note that beta 2 stimulants frequently cause
skeletal muscle tremor
72 year old woman has long-standing heart failure. Which of the following drugs has been shown to reduce mortality in chronic heart failure?
spiralactone
You are the NP treating your patient with an advanced bacterial infection. You are sure you will need dual drug therapy across multiple classes of antibiotics. While awaiting the results of your specimen culture and sensitivity, you must consider their relative potency rather than their volume of distribution.
their relative effectiveness rather than their relative potency.
You are treating a patient with peripheral neuropathy secondary to advanced diabetes. You select gabapentin because it treats neuropathic pain. With the use of gabapentin, you are aiming to address which physiologic processes associated with pain:
transduction
16 year old girl has paroxysmal attacks of rapid heart rate with palpitations and shortness of breath. These episodes occasionally terminate spontaneously but often require a visit to the emergency department of the local hospital. Her ECG during these episodes reveals an AV nodal tachycardia. Which of the following drugs would be most suitable for for prophylaxis of future episodes of acute AV nodal tachycardia?
verapamil
Patient is admitted to the emergency department after a drug overdose. He is noted to have hypotension and severe bradycardia. He has been receiving therapy for hypertension and angina. Which of the following drugs in high doses causes bradycardia?
verapamil